could you explain this?
Im a little confused, because although it would be impossible to predict the path and destination...
Remi on March 1 at 09:36PM
  • September 2006 LSAT
  • SEC1
  • Q24
1
Reply
Why A?
How is A supported in the third paragraph?
Nishant-Varma on March 1 at 09:20PM
  • September 2006 LSAT
  • SEC1
  • Q20
1
Reply
Why not D?
Is it too broad?
Nishant-Varma on February 28 at 07:31PM
  • September 2006 LSAT
  • SEC1
  • Q3
1
Reply
2 answers?
Im confused because if both R and T are in G, then that means that S cannot go into F because it ...
JohnF on February 25 at 06:13PM
  • November 2019 LSAT
  • SEC1
  • Q14
1
Reply
Doesn't A weaken?
If a company has its highest sales period during the last three months, then the company will rel...
ElleSat on February 11 at 10:55PM
  • November 2019 LSAT
  • SEC2
  • Q1
1
Reply
Villain Presentations
Please can you also explain this question? How do you know the answer is not A? The contrapos...
Anna20 on February 11 at 07:14PM
  • September 2017 LSAT
  • SEC2
  • Q21
1
Reply
could someone please explain number 19
read the responses but I'm still confused
Nativeguy on February 11 at 07:09PM
  • September 2017 LSAT
  • SEC2
  • Q19
1
Reply
Timing
On a question like this, it just so happens my work from question 2 allowed me to answer number 3...
NS1 on February 11 at 07:05PM
  • September 2017 LSAT
  • SEC2
  • Q15
1
Reply
Approach
Is there any way strategy to try and eliminate some of the options without having to test every s...
etchun on February 11 at 04:30PM
  • September 2017 LSAT
  • SEC2
  • Q2
1
Reply
Answer E?
Can you please explain why the answer is E? I originally chose A) Lowe and do not know why that ...
juliak on February 11 at 04:12PM
  • June 2017 LSAT
  • SEC4
  • Q9
1
Reply
"If substituted for the constraint " question
I'm noticing these rule substitution questions only on more recent practice exams. Is there a vid...
ritagentile on February 11 at 04:06PM
  • June 2017 LSAT
  • SEC4
  • Q6
1
Reply
Please explain
I would appreciate a break down of how to reach the correct answer and why the other answers are ...
RC on February 10 at 04:09AM
  • December 2012 LSAT
  • SEC3
  • Q25
2
Replies
Could someone please explain this?
Could someone please explain this? Thanks
jingjingxiao11111@gmail.com on February 8 at 03:42AM
  • December 1998 LSAT
  • SEC3
  • Q24
1
Reply
Please help!
Could you please explain why answer C is correct and B is wrong?
yckim2180 on February 8 at 03:41AM
  • December 1998 LSAT
  • SEC3
  • Q24
1
Reply
Please help
Why is the answer C and not D?
yckim2180 on February 8 at 03:35AM
  • December 1998 LSAT
  • SEC3
  • Q18
1
Reply
Further explain argument
Hello- I had a very difficult time understanding the argument in itself, can you please break ...
@MichaelaJ on February 8 at 03:15AM
  • November 2018 LSAT
  • SEC1
  • Q16
1
Reply
Why A is wrong and E is right
I'm just confused on why E would be the right answer, considering salaries are also increasing du...
DianaK on February 7 at 08:58PM
  • February 1994 LSAT
  • SEC1
  • Q14
2
Replies
Jury Impartiality
Why is C incorrect here? I narrowed it down to A and C. Thanks!
Anna20 on February 6 at 11:01PM
  • December 1998 LSAT
  • SEC3
  • Q2
1
Reply
Could someone please help explain this question
I completely clueless why D is the right answer. I picked E. Thanks!
liwenong28 on February 6 at 10:58PM
  • December 1998 LSAT
  • SEC1
  • Q26
1
Reply
answer choice A
Hey! I don't understand why A is not the right answer
JonathanGliboff on February 6 at 10:53PM
  • December 1998 LSAT
  • SEC1
  • Q7
1
Reply